Sei sulla pagina 1di 9

Additional Retirement Question based on new syllabus – FPSB India- Sample Paper.

Section II – Q6
A 45-year old man spends Rs. 7.5 lakh p.a., almost the amount he earns, to maintain his family. He
expects his expenses to rise by 7% p.a. He has not saved for retirement. He has a second house
which he wants to rent at Rs. 20,000 p.m. immediately, the rent expected to increase by 7 % p.a. You
advise him to create a corpus by his age of 60 by investing the rent received in an instrument yielding
9% p.a. at the end of every year. You estimate the number of years the accumulated corpus would
last taking the received rents post-retirement into account. The same is ____________
(Answer: 8 years)

Solution:

Step 1: Finding out Future value of today’s living cost at Retirement Age (60)
BGN, N= 15 (60-45), I = 7, PV= -750,000 , Fv= solve (20,69,273.656), P/y=1, C/y=1

Step 2: Finding Future Value of Ongoing Investments (Annual Rental income is getting invested till
retirement age)

As the rental income which is getting invested at the end of every year is increasing every year by
@7% p.a. So, in order to find out what amount will get accumulated till retirement age 60? , we have
to use formula of Future value of Growing Annuity.

Future value of growing annuity (FVGA) formulae for End mode calculation
Amount (1st Year Saving) * [{(1+R)^n - (1+G)^n} / (R – G)]
R – Required rate of return
G – Growth rate in salary
n- Term of Investment

240,000 (20000 pm*12) * [{(1.09)^(15) – (1.07)^(15)} / (0.09 – 0.07)]


=Rs. 1,06,01,411.03

The amount to get accumulated by retirement age 60 is Rs. 1,06,01,411 which will be utilized during
the post retirement period.

Step 3:
As per question language, the rental income from the second house will be continued during the post-
retirement period. In simple language it means, during the post-retirement period the annual rental
income will be considered as source of income to take care of living expenses.

Following steps will be considered:

Part 1: What will be the Future value of rental income at retirement age 60?

BGN, N= 15 (60-45), I=7, PV= -240,000 (20000 pm*12), Fv= solve (662,167), P/y=1, C/y=1

Part 2:

1st year expenses at retirement age 60- which will increase by


2,069,274
7% p.a
Less: 1st year rental income at retirement age 60 - which will
662,167
increase by 7% p.a
Shortfall in income to take care of expenses 1,407,107

Retirement Planning numerical Page 1


This shortfall in income of Rs. 14,07,107 on yearly basis will be withdrawn from the corpus getting
accumulated by retirement 60 in step 2.

Step 4: Calculating how long the corpus can sustain? Taking into account inflation adjusted rate of
return.
BGN, N= Solve (8 years), I= ((1.09 / 1.07)-1)*100, PV=-1,06,01,411, Pmt=14,07,107 P/y=1,
C/y=1

Section II – Q7

Mr. X who is 40 years old spends annually Rs. 7 lakh towards his household expenses. He expects to
retire at 62 years. During this period inflation is expected to be on an average 6% p.a. He wants to
cover 35 years’ living expenses for self and spouse. If the inflation in the post-retirement period
moderates to an average of 4% p.a. and he expects to generate a return of 7% from his accumulated
corpus, what corpus should he target for a comfortable retirement?
(Answer: Rs. 5.67 Crore)

Solution:

Step 1:- To calculate Future value of today’s expenses at retirement age.


BGN, N= 22 (62-40), I = 6, PV= -700,000 , Fv= solve (25,22,476.192), P/y=1, C/y=1

Step 2:- To calculate present value of all the cash flow series required during post - retirement
period, i.e from retirement age to life expectancy.

BGN, N=35, I= ((1.07/1.04)-1)*100, Pmt= -25,22,476.192, Pv= Solve (56715821.09), P/y=1,


C/y=1

Section II – Q8

An individual has recently purchased a house worth Rs. 40 lakh for self-occupation by availing housing
loan of Rs. 28 lakh at 9.25% p.a. rate of interest. The tenure of loan is 18 years. He has Rs. 12 lakh
financial assets at present. He is expected to save annually Rs. 2 lakh which he invests on a quarterly
basis beginning a quarter from now in an instrument which is expected to provide return of 9% p.a.
What would be his net worth five years from now? The value of the house which is for consumption
purposes is not considered in the net worth so arrived.
(Answer: Rs. 668,338 )

Solution:

Net worth = Total Assets – Total Liabilities.

However as per question language for the purpose of calculation (Net worth) current residential house
for self-occupation is not to be considered.

So, in order to arrive at net worth of our client five years from now following steps will be considered:

Step 1: Finding out Future value of current and ongoing investment.

END, N = 5*4, I =9, PV= -12,00,000 , PMT =-50m000 (200,000 /4) , FV= Solve (30,82,962.488) ,
P/y = 4, C/y = 1

Note: Since the quarterly investment will being after 3 month from now (i.e End of quarter) FPSB
India has considered in this question quarterly investment at the END mode.

Retirement Planning numerical Page 2


Step 2: Finding out loan outstanding amount five years from now

Part 1: Calculate EMI / Pmt of the original loan taken

End, N =18*12, I= 9.25%, PV = 28,00,000, PMT =Solve (-26,659), P/Y = 12, C/Y = 12)

Part 2: Use AMRT mode to get Balance amount after completing 5 years of loan term.

PM 1: 1 (1st Month of loan term)

PM 2: 5*12 (60th Month of loan term)

Note: Rest of the fields with regards to N, I, PV and PMT will be captured from CMPD. Which was used
to calculate EMI (PMT)?

Balance (Loan Outstanding after completing 5 years of loan term) : Solve Rs.24,14,624.306)

Step 3: Calculating Net Worth using the formulae:

Total Assets (Rs. 30, 82,962.488) – Total Liabilities (Rs. 24, 14,624.306) = Rs. 668,338.488

Section II – Q9

A 40 year old person spending Rs. 3 lakh p.a. plans to retire at age 63 and expects to live till 75
years. The basic inflation at 7% p.a. and lifestyle inflation at 1.75% p.a. are expected in the pre-
retirement period. He starts investing for retirement at Rs. 30,000 p.a. in a 10% p.a. return
instrument with immediate effect, and increases the contribution by 20% every year of the prior year
investment amount. If the expenses post-retirement are curtailed by 20%, what maximum inflation
would sustain his corpus till he survives, if the corpus is invested at 7% p.a.?
(Answer: 6.07%)

Solution:

Step 1:- To calculate Future value of today’s expenses at retirement age 63.
BGN, N= 23 (63-40), I =8.75 (7%+1.75%), PV= -300,000*80% (As, in post retirement period
expenses will be 20% less) ,Fv= solve (16,52,282), P/y=1, C/y=1

Step 2: Finding Future Value of Ongoing Investments till retirement age 63.

As the investment of Rs. 30,000 p.a which is getting invested from now onwards (BGN Mode) is
increasing every year by @ 20% p.a. So, in order to find out what amount will get accumulated till
retirement age 63? , we have to use formula of Future value of Growing Annuity.

Future value of growing annuity (FVGA) formulae for BGN mode calculation
{Amount (1st Year Saving) * [{(1+R)^n - (1+G)^n} / (R – G)]} * (1+R)
R – Required rate of return – 10%
G – Growth rate in salary – 20%
n- Term of Investment - 23 years – 1st year investment will be today at age 40 (BGN Mode) and last
investment will be at age 62 (BGN Mode). So the total investment term will be 23 years.

30,000 * [{(1.10) ^(23) – (1.20)^(23)} / (0.10 – 0.20)]


=Rs. 1,71,87, 921 * (1.10)
= Rs. 1,89,06, 713

The amount to get accumulated by retirement age 63 is Rs.1,89,06, 713which will be utilized during
the post retirement period.

Retirement Planning numerical Page 3


Step 3:- As per question language “what maximum inflation would sustain his corpus till he survives,
if the corpus is invested at 7% p.a.?”

For this we will first calculate inflation adjusted rate of return (i.e use step 2 of retirement planning
calculation) and from the stated ANSWER we will calculate Inflation rate.

Part 1:
BGN, N=12 (75-63), I= Solve (0.876560), Pmt= -16,52,282, Pv= 1,89,06, 713 P/y=1, C/y=1

The “I” (0.876560%) indicates inflation adjusted rate of return.

Part2:
Inflation Adjusted rate of return = (1+ Return) / (1+ Inflation rate)

Inflation Adjusted rate of return = 0.876560 % = 1.00876560


Return during post retirement period = 7% = 1.07
Inflation Rate = To solve

(1.00876560) = 1.07 / (1+ Inflation rate)

Inflation = 6.07%

Section III – Q4

A 55 year old individual could not save for retirement while he met his children’s education and
marriage. In the next 5 years he would have Rs. 35,000 per month investible surplus. The couple has
household expenses of Rs. 27,000 per month. They have a second house which earns Rs. 18,000 per
month in rental income. You direct their monthly savings to an 8 % p.a. instrument. Considering this
return as sustainable after retirement also, inflation at 6% p.a., rental increments at 6% p.a., you
estimate the period after retirement at 60 years when they may have to start curtailing their
expenses. The same is ____.

Solution:

Step 1:- To calculate Future value of today’s expenses at retirement age 60.

BGN, N=5 (60- 55), I=6, PV=-27000, Fv= Solve (36,132), P/y=1, C/y=1

Step 2: Finding out Future value of ongoing investment.

BGN, N=5*12, I= 8, Pmt= -35000 (Monthly Saving available), Fv= Solve (25,69,489.845), P/y=12,
C/y=1

Step 3:
As per question language, the rental income from the second house will also continue during the post-
retirement period. In simple language it means, during the post-retirement period the annual rental
income will be considered as source of income to take care of living expenses.

Following steps will be considered:

Part 1: What will be the Future value of rental income at retirement age 60?

BGN, N= 5 (60- 55), I=6, PV= -18,000, Fv= solve (24,088), P/y=1, C/y=1

Retirement Planning numerical Page 4


Part 2:

1st month expenses at retirement age 60- which will increase


36,132
by 6% p.a
Less: 1st month rental income at retirement age 60 - which
24,088
will increase by 6% p.a
Shortfall in income to take care of expenses 12,043
This shortfall in income of Rs. 12,043 on monthly basis will be withdrawn from the corpus getting
accumulated by retirement 60 in step 2.

Step 4: Calculating how long the corpus can sustain? Taking into account inflation adjusted rate of
return.
BGN, N= Solve (259) (Which indicates 21 years and 7 months), I= ((1.08/1.06)-1)*100,
PV= -25,69,489.245, Pmt= 12044, P/y=12, C/y=1

Section III – Q6

A 30 years old lady plans to retire at 50. She already has financial investments amounting to Rs. 18
lakh. Her current expenses are Rs. 27,000 per month. She wants to cover 30 years post-retirement
expenses inflation-adjusted by a suitable annuity. Average inflation is considered at 7% p.a. and the
annuity is expected to yield 8% p.a. You advise to invest her current financial investments at 9.5%
p.a. If she can incrementally invest Rs. 2.5 lakh annually, you optimize average rate of return to
achieve this goal. The same is
(Answer: 12.61%)

Solution:

Step 1: FV of today monthly expense at the time of retirement


BGN, N=20, I=7, PV=-27000, Fv= Solve (104,481), P/y=1, C/y=1

Step 2: PV of the entire cash flow requirement during the post - retirement phase.
BGN, N=30*12 (As stated in the question, to provide 30 years post-retirement expenses inflation-
adjusted”), I= ((1.08/1.07)-1)*100, PV= Solve (3,28,33,627), Pmt= 104,481, P/y=12, C/y=1

Step 3: FV of current investment.

Note: As per question, “Current investment amount of Rs. 18.00 Lakhs is allocated towards
Retirement goal” and is growing @ 9.25% p.a

BGN, N=20, I=9.25,Pv= -18,00,000, Fv= Solve (11054901),P/y=1, C/y=1

Step 4: Finding out rate of return for regular investment

Corpus required during post retirement period (Step 2) 3,28,33,627


Less: Future Value of Current investment (Step 3) 1,10,54,901
Shortfall 2,17,78,726

The shortfall amount of Rs. 2.17 crore will be accumulated with the help of regular yearly investment
of Rs.2.50 Lakhs during the accumulation period, i.e from age 30 to age 50.

Retirement Planning numerical Page 5


So we have to find out, at what investment rate Mrs. A will be able to accumulate the shortfall
amount, if she starts investing Rs.2.50 Lakhs on yearly basis for next 20 years.

BGN, N=30, I= Solve (12.61), Pmt=- 250,000, Fv= 32833627 - 11054901,P/y=1, C/y=1

Section III – Q8

Mr. A purchased a flat worth Rs. 50 lakh in January 2007 by availing a housing loan of Rs. 35 lakh for
tenure 15 years at the rate of 9% p.a. The value of his flat as in January 2013 has appreciated to Rs.
90 lakh. What approximate value of home equity can he consider in his flat towards his unencumbered
interest after also setting aside 15% of the appreciation value towards taxes and other costs to be
discharged on selling the unit?
(Answer: Rs.57,78,751 )

Note:
1. The word unencumbered means "Asset or property that is free from debt, and clear of
any legal defect in its title and, therefore, can be easily sold or mortgaged
2. Home Equity is nothing but “current market of the property minus Loan outstanding “

Step 1: Calculate EMI / Pmt of the original loan taken


Set= End, n =15*12, I= 9%, PV = 35,00,000, PMT =Solve (35499.33), P/Y = 12, C/Y = 12)

Step 2: Use AMRT mode to get Balance amount as on 1st Jan 2013

PM 1: 1
PM 2: 6*12 (Jan 2007 to 1st Jan 2013)

Note : Rest of the fields with regards to N,I, PV and PMT will be captured from CMPD. Which was used
to calculate EMI (PMT)?

Balance: Solve (26,21,248.607)

Step 3: Gain in property price and tax on it

Rs.90,00,000 (current value) -Rs.50,00,000 (cost of purchase)


=Rs.40,00,000* 15% (Tax rate)
=Rs.600,000

Step 4: Value in hand


Current Value- Loan Outstanding -Tax amount
=Rs.90,00,000 - Rs.26,21,248Lakhs - Rs. 600,000 =Rs.57,78,752

Retirement Planning numerical Page 6


Section III – Q9

Mr. A has invested annually Rs. 2 lakh towards his retirement in an aggressive fund from his age of 40
onwards. After initial high returns, the fund could generate return of just 3.5% p.a. in 10 years. He
can direct a higher amount towards retirement goal in the remaining 10 years to retirement. You
advise to switch half of the accumulated funds along with fresh investment in a debt fund with
indicative return of 8% p.a. in the future. To achieve a target corpus is Rs. 1.2 crore, what revised
amount should be invested every year if the future expectation from aggressive fund is 11% p.a.?
(Answer: Rs. 3.79 Lakhs)

Solution:

Step 1: Current value of regular investments of Rs.2.00 Lakhs p.a of last 10 years growing @ 3.5%.

BGN, N=10, I=3.5, Pmt= -200000, Fv= Solve (24,28,398), P/y=1,C/y=1

Step 2: Rebalancing of the portfolio


As per question language, for the remaining period to retirement (i.e 10 years), the accumulated
funds in Step 1 will be rebalanced between Equity and Debt in the equal ratio, (ie 50:50)

Rebalancing of the portfolio value


Debt = 24,28,398 * 50% = 12,14,199
Equity= 24,28,398 * 50% = 12,14,199

Step 3: Future value of equity investment.

As per question, equity portion of the portfolio, ie Rs. 12,14,199, will continue to grow @ 11% p.a
without any regular investment till the remaining period to retirement (10 years from now)

BGN, N=10, I=11, Pv=-12,14,199, Fv= Solve (34,47,622), P/y=1, C/y=1

Step 4: Regular Investment in Debt

Corpus required at retirement age (As per question) 1,20,00,000


Less: Future Value of Equity portion of investment (Step 3) 34,47,622
Shortfall 85,52,377

The shortfall amount of Rs. 85,52,377 will be accumulated with the help of regular yearly investment
in Debt Fund portfolio during the accumulation period, i.e from age 40 to age 50

So we have to find out, regular investments to be done on yearly basis in Debt Fund taking into
account the current value available in Debt Fund after rebalancing the portfolio (Step 2) .Investment
rate for Debt Fund is 8% p.a.

BGN, N=10, I=8, PV= -12,14,199, Fv=85,52,377 (Shortfall Amount), Pmt= Solve (379,087),
P/y=1, C/y=1

Retirement Planning numerical Page 7


Section IV- Q6

A person invested Rs. 45 lakh in a 30-year fixed monthly annuity providing a yield of 9% p.a. What will
be the amount of monthly annuity if the start date is deferred by 3 years?
(Answer: Rs.45, 100)

Solution:

Step 1:
As the requirement of monthly annuity will commences after 3 years from. We will first find out FV of
current investment amount growing @ 9% p.a

BGN, N=3, I=9, Pmt= -45,00,000, FV= Solve (5827630.5), P/y=1,C/y=1

Step 2:
Finding out regular monthly Annuity to earn for next 30 years from the corpus generated in Step 1

BGN, N=30*12, I=9, Pmt=Solve (45,100), PV= -58,27,630.5, P/y=12,C/y=1

Retirement Planning numerical Page 8


Section IV- Q7

A person at age 57 has accumulated Rs. 50 lakh towards retirement funds and opts for premature
retirement. He purchases an immediate annuity for a total term of 20 years, a fixed monthly amount for
the initial period of 10 years and a provision to double the monthly amount in the second 10-year
period. If the minimum yield guaranteed in the annuity is 8% p.a., what monthly amount he is
expected to receive in the subsequent 10-year period?
(Answer - Rs. 61,821)

Solution:
In order to calculate the expected annuity to receive in the subsequent 10-year period, we will
do reverse calculation. For this we take following assumption.

Step 1:
For the 1st Ten year if you are likely to receive monthly annuity (PMT) of Rs. 100, then what is
the present value (Today) of the entire cash flow series discounted @ 8%.

BGN, N=10*12, I=8, Pmt= 100, PV= Solve (-8,397), P/y=12, C/y=1

Step 2:
In the subsequent term of Ten year period, if you are likely to receive monthly annuity (PMT) of
Rs. 200 (Annuity amount will be doubled), then what is the present value of the entire cash flow
series discounted @ 8%.

BGN, N=10*12, I=8, Pmt= 200, PV= Solve (-16,793), P/y=12, C/y=1

Step 3:
The PV of Rs. 16,793 is value as on Beginning of second 10 year period which will be further
discounted @ 8% to get value as on Today’s date.

BGN, N=10, I=8, FV= 16793, PV= Solve (-7,779), P/y=12, C/y=1

Step 4:
Total value of investment or rather lump sum amount required to invest Today in order to earn
monthly annuity of Rs.100 in the 1st 10 year and Doubled annuity of Rs. 200 in the subsequent 10
year period is Rs. 16,176 (8397 + 7779).

Then with Lump sum investment of Rs. 50.00 Lakhs today, how much annuity can be earned on
monthly basis? Will be as follow:

Amount Invested Annuity to earn


16,176 100
Solve (Rs. 30911)
50,00,000
(50,00,000 * 100) / (16176)

The answer or annuity amount of Rs. 30,911 indicates monthly annuity to be earned during 1st 10year
period.

So as per question language, the annuity amount to be earned in the subsequent 10 year period will
be doubled. Then annuity amount in subsequent 10 year period will be Rs. 61,822 {30911 * 2).

Retirement Planning numerical Page 9

Potrebbero piacerti anche